LSAT and Law School Admissions Forum

Get expert LSAT preparation and law school admissions advice from PowerScore Test Preparation.

User avatar
 KelseyWoods
PowerScore Staff
  • PowerScore Staff
  • Posts: 1079
  • Joined: Jun 26, 2013
|
#81951
Hi grunerlokka!

Yes, the conclusion of this argument would be the first sentence that "Consumers will be hurt by the new lower ceilings on halibut catches." The second sentence supports the first sentence by giving us the reason why consumers will be hurt by the lower ceilings: "these restrictions are likely to result in an increase in the price of the fish."

This is a sneaky question stem, though, because instead of strengthening the conclusion of the argument, they're asking us specifically to strengthen "the claim that the price of halibut will increase." So we end up needing to focus on finding an answer choice that will support that last sentence.

Hope this helps!

Best,
Kelsey

Get the most out of your LSAT Prep Plus subscription.

Analyze and track your performance with our Testing and Analytics Package.